Zbadaj monotoniczność i ekstrema

Różniczkowalność, pochodna funkcji. Przebieg zmienności. Zadania optymalizacyjne. Równania i nierówności z wykorzystaniem rachunku różniczkowego.
monisia8062
Użytkownik
Użytkownik
Posty: 107
Rejestracja: 18 gru 2011, o 12:52
Płeć: Kobieta
Lokalizacja: Ruda Śląska
Podziękował: 17 razy

Zbadaj monotoniczność i ekstrema

Post autor: monisia8062 »

Zbadać monotoniczność i ekstrema oraz narysować wykres funkcji \(\displaystyle{ y=\left| \left| f\left( x\right) \right| \right|}\)
jeżeli
\(\displaystyle{ f\left( x\right) = \lim_{n \to \infty } \left( x-x ^{4}+x ^{7} -...+ \left( -1 \right) ^{n+1} \cdot x ^{3n-2} \right)}\)
Ostatnio zmieniony 5 mar 2012, o 16:59 przez Afish, łącznie zmieniany 1 raz.
Powód: Niepoprawnie napisany kod LaTeX-a. Proszę zapoznaj się z http://matematyka.pl/178502.htm .
leapi
Użytkownik
Użytkownik
Posty: 622
Rejestracja: 4 mar 2012, o 07:53
Płeć: Mężczyzna
Lokalizacja: PL
Podziękował: 1 raz
Pomógł: 86 razy

Zbadaj monotoniczność i ekstrema

Post autor: leapi »

a wzór \(\displaystyle{ S_n=\frac{a_1}{1-q}}\) dla \(\displaystyle{ |q|<1}\) jest znany??
monisia8062
Użytkownik
Użytkownik
Posty: 107
Rejestracja: 18 gru 2011, o 12:52
Płeć: Kobieta
Lokalizacja: Ruda Śląska
Podziękował: 17 razy

Zbadaj monotoniczność i ekstrema

Post autor: monisia8062 »

Czyli mam obliczyć :
\(\displaystyle{ \lim_{ x\to \infty }\left( \frac{x}{1+x ^{3} } \right)}\) ?
leapi
Użytkownik
Użytkownik
Posty: 622
Rejestracja: 4 mar 2012, o 07:53
Płeć: Mężczyzna
Lokalizacja: PL
Podziękował: 1 raz
Pomógł: 86 razy

Zbadaj monotoniczność i ekstrema

Post autor: leapi »

Pod warunkiem,że \(\displaystyle{ |q|<1}\)

-- 5 mar 2012, o 19:20 --
monisia8062 pisze:Zbadać monotoniczność i ekstrema oraz narysować wykres funkcji \(\displaystyle{ y=\left| \left| f\left( x\right) \right| \right|}\)
jeżeli
\(\displaystyle{ f\left( x\right) = \lim_{n \to \infty }}\)
monisia8062 pisze:
\(\displaystyle{ f\left( x\right) = \lim_{n \to \infty }}\)
[/quote]
monisia8062 pisze: : \(\displaystyle{ \lim_{ x\to \infty } ?}\)

Która jest poprawna
monisia8062
Użytkownik
Użytkownik
Posty: 107
Rejestracja: 18 gru 2011, o 12:52
Płeć: Kobieta
Lokalizacja: Ruda Śląska
Podziękował: 17 razy

Zbadaj monotoniczność i ekstrema

Post autor: monisia8062 »

Tak jak napisałam na samym początku
leapi
Użytkownik
Użytkownik
Posty: 622
Rejestracja: 4 mar 2012, o 07:53
Płeć: Mężczyzna
Lokalizacja: PL
Podziękował: 1 raz
Pomógł: 86 razy

Zbadaj monotoniczność i ekstrema

Post autor: leapi »

\(\displaystyle{ f(x)=\frac{x}{1+x^3} \ \ \ \ \ \ dla \ \ \ \ \ \ |x^3|<1 \Leftrightarrow x \in(-1,1)}\)
ODPOWIEDZ